CFP Class #3

Lakukan tugas rumah & ujian kamu dengan baik sekarang menggunakan Quizwiz!

Harold is covered by a $180,000 group term life insurance policy, and his daughter is the beneficiary. Harold's employer pays the entire cost of the policy, for which the uniform annual premium is $8 per $1,000 of coverage. How much of this premium is taxable to Harold, assuming the coverage is provided for the entire year? $0 $1,040 $1,440 $640

1,040 The cost of up to $50,000 of coverage is nontaxable. $180,000 − $50,000 = $130,000 ($130,000 ÷ $1,000) × $8 = $1,040 taxable coverage

Robert receives incentive stock options (ISOs) with an exercise price of $16 when the stock is trading at $16. Robert exercises these options 2 years after the date of the grant when the stock price is $37 per share. Which of the following statements is correct? 1. Upon exercise, Robert will have no income for regular tax purposes. 2. Robert will have W-2 income of $21 per share upon exercise. 3. Robert will have $16 of alternative minimum taxable income (AMTI) upon exercise. 4. Robert's adjusted basis for regular income tax will be $37 at exercise.

1. Upon exercise, Robert will have no income for regular tax purposes. Robert does not have regular income at the date of exercise. Robert's adjusted basis will be $16. The alternative minimum tax (AMT) adjustment will be the difference between the fair market value and the exercise price, or $37 − $16 = $21.

Faith has a small lot used in her business. She exchanges it for a like-kind asset, another small lot, owned by Angela. At the time of the exchange, the basis of Faith's asset was $20,000. She gave Angela $30,000 cash plus the asset in exchange for her asset, which was worth $36,000. Angela's basis in her original asset was $5,000. What is Angela's gain or loss? A) $30,000 gain recognized due to Angela's receipt of boot B) $31,000 gain realized and recognized C) $0 gain recognized D) $14,000 loss recognized

A) $30,000 gain recognized due to Angela's receipt of boot. After 2017, only real property qualifies for Section 1031 exchange treatment.

Which of the following statements are correct regarding the taxation of fringe benefits? I. The value of the fringe benefit is included in the employee's gross income unless the Code specifically excludes it from taxation. II. The value of the fringe benefit is excluded from the employee's gross income unless the Code specifies otherwise. III. The value of the fringe benefit is taxable if the benefit is only provided to employees owning more than five percent of the company and if the fringe benefit has a nondiscrimination requirement. IV. The value of the fringe benefit is always taxable if someone other than the employee (such as the employee's spouse) benefits from the fringe benefit provided by the employer.

B. I and III The value of the fringe benefit is included in the employee's gross income unless the Code specifically excludes it from taxation. Someone who owns more than five percent of a company is a highly compensated employee and/or key employee. If nondiscrimination requirements apply, fringe benefits that are only provided to this group will be taxable even if they would be excluded from taxation for other employees.

Harold is covered by a $170,000 group term life insurance policy, and his daughter is the beneficiary. Harold's employer pays the entire cost of the policy, for which the uniform premium is $1.27 per $1,000 of coverage per month. How much of this premium is taxable to Harold, assuming he is covered for 12 months? A) $0 B) $152 C) $1,829 D) $2,591

C) $1,829 The correct answer is (C). The cost of up to $50,000 of coverage is nontaxable. $170,000 − 50,000 = $120,000 [($120,000 ÷ $1,000) × $1.27] x 12 months = $1,829 taxable coverage

Hank is covered by a group term life insurance policy that provides coverage of $225,000. Hank's employer pays the entire cost of the policy, for which the uniform premium is $0.23 per $1,000 of coverage per month. How much of this premium is taxable to Hank, assuming he is covered for 12 months? A) $828 B) $69 C) $40 D) $483

D) $483

Ryan owns an apartment building. Real estate prices in the area have been rising, and he thinks he could make a profit by selling the building. He originally purchased the building for $1,200,000 and took depreciation deductions of $700,000. Straight-line depreciation would have been $500,000. What are the tax consequences if he sells the building for $3,500,000? A) He will have ordinary income of $200,000. B) He will have $700,000 of unrecaptured Section 1250 gain. C) He will have capital gains of $3,000,000. D) None of the above applies to his situation.

The correct answer is (A). $3,500,000 Sale Price − 500,000 Basis $3,000,000 Gain (Total) 200,000 Ordinary Income − 500,000 25% Gains $2,300,000 Long-Term Capital Gain (LTCG) Of the $3,000,000 gain, $200,000 is recognized as ordinary income since this represents the Section 1250 depreciation recapture (excess depreciation of $700,000 above the straight-line depreciation of $500,000), $500,000 is recognized as unrecaptured Section 1250 gain, and the remainder is considered a long-term capital gain under Section 1231.

Colin, a single taxpayer, received $17,000 of Social Security benefits this year. He also received $14,000 of interest income. How much of Colin's Social Security benefits must be included in his gross income this year? A) $0 B) $8,500 C) $14,450 D) $17,000

The correct answer is (A). Since the total of Colin's MAGI and one-half of his Social Security benefits is less than the base amount, none of his Social Security benefits are included in gross income. $14,000 + (0.50 × $17,000) = $22,500 < $25,000

In the current year, Frank, who is not a real estate professional, invested $25,000 for a 25 percent interest in a real estate rental partnership in which he is a general partner and a material participant. Frank's adjusted gross income (AGI) was $125,000, and his allocated loss from the real estate activity was $30,000. Frank has no passive income this year. What is Frank's deductible loss on his federal tax return from the above transaction for the current year? A) $12,500 B) $15,000 C) $25,000 D) $30,000

The correct answer is (A). The maximum deductible real estate loss is $25,000 per year. This maximum loss is phased out for AGIs of $100,000 to $150,000. Since Frank's AGI was $125,000, the maximum loss he can deduct is $12,500, calculated as follows: $125,000 − $100,000 = $25,000 $25,000 × 0.5 = $12,500 $25,000 − $12,500 = $12,500 allowable loss

Gaia owned an office building in Roswell that she rented out to several production companies. The building was destroyed by a fire and was a complete loss. Gaia received a settlement from her insurance company during the same year as the fire and would like to reinvest in a new property. Gaia wants to make sure that she is eligible for involuntary conversion treatment. Which of the following is NOT correct regarding the requirements for involuntary conversion treatment on Gaia's transaction? A) Because Gaia rented out the building instead of using the property directly, the replacement property must meet the functional use test. B) Gaia must invest the proceeds in a replacement property that has a similar use to the property that was destroyed in the fire. C) Gaia must reinvest the insurance proceeds within 2 years from the end of the year in which the fire occurred. D) Since Gaia received cash as a result of the involuntary conversion, nonrecognition treatment is not mandatory even if she meets all of the requirements.

The correct answer is (A). The replacement property must meet the taxpayer use test, not the functional use test, since Gaia did not use the property directly. The taxpayer use test requires replacement property to be used by the taxpayer in an activity which is treated in the same way for tax purposes in order to qualify for nontaxable exchange treatment. All of the other statements regarding the nontaxable exchange treatment of Gaia's transaction are true. Reinvestment must be within 2 years of realization, not the event, and is always December 31. Because the proceeds were received in the year of the fire, the replacement period is 2 years from that year.

Alistair has real estate used in his business. He exchanges it for like-kind real estate owned by Pablo. (Alistair and Pablo are not related.) The basis of Alistair's real estate is $50,000, and he gives Pablo $10,000 cash plus the real estate in exchange for Pablo's real estate, which is worth $46,000. Pablo's basis in his original real estate is $10,000. What is Alistair's recognized gain or loss? A) $0 B) $11,000 gain C) $14,000 loss D) $14,000 gain

The correct answer is (A). The values exchanged must be equal. Alistair gives his property plus $10,000 in exchange for a property worth $46,000 therefore Alistair's property is valued at $36,000. Alistair has no gain, rather he has an increase in basis.

Alejandro, aged 54 and single, earns a salary of $180,000 working for a manufacturing company. He is an avid saver, and over the years he has amassed an investment portfolio of $2,000,000. He expects the portfolio to appreciate in value at an average rate of 8 percent per year. Last year, he received dividends and interest of $40,000 from the portfolio. After speaking with a financial planner, Alejandro decided to invest $30,000 for a 15 percent interest in a passive activity in the current year. Operations of the activity resulted in a loss of $300,000, $45,000 of which was Alejandro's share. How is his loss for the current year characterized? A) $15,000 is suspended under the at-risk rules, and $30,000 is suspended under the passive-loss rules. B) $30,000 is suspended under the at-risk rules, and $15,000 is suspended under the passive-loss rules. C) $45,000 is suspended under the passive-loss rules. D) $45,000 is suspended under the at-risk rules.

The correct answer is (A). $15,000 of Alejandro's loss is suspended under the at-risk rules, leaving a potential deduction of $30,000. The $30,000 loss is suspended under the passive-loss rules.

Arthur owns a life insurance policy that is a modified endowment contract. He recently reassessed his insurance needs and decided that he would like to exchange his current policy for a different insurance product. Which of the following transactions might result in gain realization and recognition? A) Arthur trades his modified endowment contract for a permanent life insurance policy. B) Arthur trades his modified endowment contract for a different modified endowment contract. C) Arthur trades his modified endowment contract for an annuity. D) None of the above transactions would result in the realization and recognition of gain.

The correct answer is (A). If Arthur trades his modified endowment contract for a life insurance policy, the transaction will not be eligible for the deferral of gain under Section 1035.

Anne, a single taxpayer, has an alternative minimum taxable income (AMTI) of $725,000. Assuming the AMT exemption phaseout starts at $523,600, by what amount is her alternative minimum tax (AMT) exemption reduced this year? A) $0 B) $73,600 C) $23,250 D) $50,350

The correct answer is (D). Because Anne's AMTI is above the AMT phaseout threshold amount, her AMT exemption must be reduced. Anne's exemption is reduced by 25 percent of the amount that her AMTI exceeds $523,600 (the threshold). Therefore, her exemption must be reduced by $50,350, that is, ($725,000 − $523,600) × 0.25.

Last year, Jacques paid the following interest: $7,300 on a home mortgage $1,000 on a home equity loan used to purchase furniture for a personal residence $1,800 on a loan used to purchase State of Louisiana general purpose bonds If Jacques itemizes his deductions for last year, what is the amount of deductible interest expense? A) $7,300 B) $8,300 C) $9,100 D) $10,100

The correct answer is (A). Only the interest on the home mortgage is deductible. The interest on the home equity loan used to purchase household furniture is nondeductible based on the Tax Cuts and Jobs Act of 2017 (TCJA). The interest on the loan to purchase State of Louisiana bonds is not deductible because he used the loan to purchase tax-exempt municipal bonds.

Kayla was recently diagnosed with lung cancer and was certified by her doctor on June 1st of the current year as terminally ill. On July 1st of the current year, Kayla sold her life insurance policy with a face value of $500,000 for $340,000. Assuming she paid $50,000 in premiums, how much of the $340,000 proceeds must she include in her gross income for the current year? A) $0 B) $34,000 C) $50,000 D) $290,000

The correct answer is (A). Option (A) is correct because Kayla will have no inclusion in her gross income. If an individual is terminally ill (expected to die within 24 months), accelerated death benefits are not taxable.

Which individual can make a deductible contribution to a traditional IRA in the current year? A) Phil, who is married, has an adjusted gross income (AGI) of $155,000, and his spouse is an active participant in her employer's defined-contribution plan. However, Phil is not an active participant. B) Kyle, who is single, has an AGI of $85,000 and is an active participant in his employer's defined-benefit plan. C) Claire, who is married, has an AGI of $135,000 and is an active participant in her employer's defined-contribution plan. D) Kim, who is single, has no earned income.

The correct answer is (A). Option (A) is correct because although Phil's spouse is an active participant, Phil is not. Therefore he may make a deductible contribution to a traditional IRA as long as their joint AGI does not exceed $214,000 in 2022. Option (B) is incorrect because Kyle's AGI exceeds the phase-out range for single individuals who are active participants. Option (C) is incorrect because Claire's AGI exceeds the phase-out range for married individuals who are active participants. Option (D) is incorrect because Kim must have earned income in order to contribute to a traditional IRA.

Over the past 4 years, Aliah, aged 28, has contributed a total of $20,000 to a Roth IRA. The current balance is $25,000. She was tired of renting, so this year she took a distribution of $15,000 for a down payment on a home. What amount of the distribution should she include in her gross income this year? A) $0 B) $3,000 C) $5,000 D) $15,000

The correct answer is (A). Roth IRA contributions can be distributed at any time without including the distribution in gross income.

Colin, a single taxpayer, received $17,000 of Social Security benefits this year. He also received $14,000 of interest income. How much of Colin's Social Security benefits must be included in his gross income this year? A) $0 B) $8,500 C) $14,450 D) $17,000

The correct answer is (A). Since the total of Colin's MAGI and one-half of his Social Security benefits is less than the base amount, none of his Social Security benefits are included in gross income. $14,000 + (0.50 × $17,000) = $22,500 < $25,000

Hector, who is single, is a partner in a financial planning firm with one other individual. Hector's share of partnership income is $115,000, and his total taxable income (excluding the qualified business income deduction) is $140,000, placing him in the 24 percent marginal tax bracket. What is Hector's adjusted marginal tax rate on the partnership income after factoring in the Section 199A qualified business income (QBI) deduction? A) 19.2% B) 24% C) 4.8% D) 22%

The correct answer is (A).Hector is in the 24 percent bracket, and his income is below the phase-out range. The net effect of the qualified business income deduction will reduce the marginal tax rate on his business income to 19.2 percent, or 24% × (1 − 0.20).

Troy is starting a new business. His major concern is to limit his risk and his liability, but he is also concerned with the ability of the company to raise capital. At some point, he would like to be able to sell partial interests in the business to other investors or perhaps to even take the company public, so he does not want to limit the number of potential owners. Which of the following entities would best suit Troy's needs? A) A C corporation B) An S corporation C) An LLC D) A general partnership

The correct answer is (A).Option (B) is incorrect because Troy does not want to limit the number of potential owners and an S corporation cannot have more than 100 owners. Option (C) is incorrect; if Troy is interested in taking the company public in the future, the LLC form would not be appropriate. Option (D) is incorrect because a partnership would not provide the limited liability in which he is interested. Only a C corporation meets all of Troy's qualifications.

Edgar is starting a new business with his best friend, Nico. They are concerned about liability. They would prefer flow-through taxation because they expect to have losses in the first few years. They are not concerned about incurring self-employment taxes. Which of the following entities would best suit their needs? A) An LLC B) A C corporation C) A proprietorship D) A general partnership

The correct answer is (A).Option (B) is not correct because Edgar and Nico want flow-through taxation. Option (C) is not correct because proprietorships do not allow multiple owners. Option (D) is incorrect because a partnership would not provide the limited liability that Edgar and Nico are looking for. Only the LLC meets all of their requirements.

Gina and Haley are starting a new business. They are concerned about liability. They would like to have flow-through taxation. They will own the company 50/50 but want to allocate the profits 60/40. Which of the following entities would best suit their needs? A) An LLC B) An S corporation C) A C corporation D) A partnership

The correct answer is (A).Option (C) is not correct because a C corporation would not provide flow-through taxation. Option (D) is not correct because a partnership would not provide limited liability. Options (A) and (B) both provide flow-through taxation and limited liability; however, an S corporation does not allow the allocation of income to be different than ownership, whereas an LLC would allow this.

In a transaction for the sale of a bank building at an amount in excess of the original purchase price, any previously taken straight-line depreciation is A) recaptured as ordinary income. B) treated as capital gain under regular capital-gain rules. C) ignored for the calculation of capital gains. D) treated as capital gain taxed at 25 percent.

The correct answer is (D). Straight-line depreciation on Section 1250 assets is taxed at 25 percent.

Kyle, Scott, and Andrew wish to start a manufacturing business. The business is expected to generate substantial income, which it will reinvest for many years. Kyle has assets which he plans to contribute to the business. Kyle is also concerned about showing too much business income on his personal income tax return. Which of the following entity structures would serve as an appropriate form of organization to satisfy Kyle, Scott, and Andrew's objective? I. A limited partnership with Kyle as the limited partnerII. A business trust with all three as equal interestsIII. An S corporation with all three as equal shareholdersIV. A C corporation with all three as shareholders A) IV only B) I and IV C) II and IV D) I, II, and III

The correct answer is (A).Statements I, II, and III are all pass-through entities, so the owners would need to report pro rata on their personal income tax return. Therefore statements I, II, and III are incorrect. The C corporation will not force the shareholders to distribute their respective shares of corporate income, although there may be additional taxes and penalties imposed on those C corporations who choose not to distribute their earnings once certain accumulated earnings thresholds are exceeded.

Daniel and Samantha were divorced. Their only marital property was a personal residence with a value of $300,000 and cost of $125,000. Under the terms of the divorce agreement, which did not include the word "alimony," Samantha would receive the house. She would pay Daniel $20,000 each year for 5 years. If Daniel died before the end of the 5 years, the payments were to be made to his estate. Samantha and Daniel lived apart when Daniel received the payments. Which statement is true about this situation? A) Daniel does not recognize any income from the above transaction. B) Daniel must recognize an $87,500, that is, ½ × ($300,000 − $125,000), gain on the sale of his interest in the house. C) Samantha can deduct $20,000 a year for alimony paid. D) Samantha can deduct $25,000 as alimony paid.

The correct answer is (A).The payments are not alimony because the payments would continue after the death of the payee.

Jackson owns a rental home in Florida. He decided that he would like to acquire a rental home in Colorado. Kade has a rental home in Colorado. For health purposes, Kade must relocate to Florida. Jackson and Kade decide to exchange properties under Section 1031 of the code. Kade's basis in his property is $100,000, and Jackson's basis in his property is $75,000. Jackson and Kade exchange the two properties, but Kade needs to give Jackson an additional $25,000 in cash. The fair market value (FMV) of Kade's property is $100,000, and the FMV of Jackson's property is $125,000. What is Jackson's recognized gain? A) $0 B) $25,000 C) $50,000 D) $75,000

The correct answer is (B).

Five years ago, the Joe Jones company purchased a bulldozer for $200,000. Since the purchase, they have taken $62,000 in straight-line depreciation. They now have an offer to sell the bulldozer for $145,000. Assuming they sell the bulldozer, what is the tax consequence? A) They have a $7,000 long-term capital gain (LTCG). B) They have $7,000 in ordinary income. C) They have a long-term capital loss (LTCL) of $55,000. D) There are no tax consequences.

The correct answer is (B). $200,000 Basis − 62,000 Depreciation $138,000 Adjusted Basis $145,000 Sales Price − $138,000 Adjusted Basis $7,000 Gain (Ordinary Income) The character of the gain is ordinary due to the recapture of depreciation as ordinary income.

Assume a taxpayer incurred business losses in Year 1 and generated an NOL of $50,000 that would be carried forward to offset income in future years. Assuming the taxpayer had $20,000 of taxable income and incurred no additional business income or loss in Year 2, what amount of NOLs could be recognized in Year 2? A) $0 B) $16,000 C) $20,000 D) $50,000

The correct answer is (B). NOLs can only be carried forward and are limited to 80 percent of taxable income. In this case, 80 percent of taxable income ($20,000) is equal to $16,000. Therefore, only $16,000 of the NOL carryforward ($50,000) can be recognized in Year 2.

Teddy has three capital transactions for the current year: A short-term capital loss (STCL) of $8,000 A short-term capital gain (STCG) of $6,000 A long-term capital loss (LTCL) of $5,000 What is the net effect on Teddy's income tax this year if he is in the 37 percent income tax bracket? A) A $2,000 deduction B) A $1,110 tax reduction C) A $2,590 tax reduction D) A $7,000 deduction

The correct answer is (B). Net the STCG and STCL. $6,000 + -$8,000 = -$2,000 STCL $5,000 (LTCL) + $2,000 (STCL) = $7,000 total loss Teddy can only use $3,000 of losses this year to offset ordinary income at 37 percent. $3,000 × 0.37 = $1,110 The remaining $4,000 is a long-term capital-loss carryover. LTCG 0 STCG $6,000 LTCL + $5,000 STCL + -$8,000 LTCL $5,000 NSTCL $2,000 Do not net.

Eric has realized several capital gains and losses this year: $10,000 in short-term capital gains $15,000 in short-term capital losses $18,000 in long-term capital gains $6,000 in long-term capital losses What is Eric's net gain or loss? A) $7,000 net short-term capital gain B) $7,000 net long-term capital gain C) $7,000 net long-term capital loss D) $7,000 net short-term capital loss

The correct answer is (B). STCG $10,000 LTCG $18,000 STCL + -$15,000 LTCL + -$6,000 STCL $5,000 LTCG $12,000 Net LTCG $7,000

Kenneth completed his freshman year of college and his parents paid $6,000 in qualified education expenses this year. Kenneth's parents are married-filing-jointly (MFJ) taxpayers and had an adjusted gross income (AGI) of $175,000. What, if any, education credit will provide them with the highest credit, and how much is that credit? A) Kenneth's parents can claim an American Opportunity tax credit in the amount of $1,875. B) Kenneth's parents can claim an American Opportunity tax credit in the amount of $625. C) They are not eligible to claim any education tax credits since their AGI exceeds the phaseout. D) Kenneth's parents can claim an American Opportunity tax credit in the amount of $2,500.

The correct answer is (B). Since Kenneth's parents' AGI falls in between the American Opportunity tax credit phaseout (that is, $160,000 to $180,000), his parents are entitled to claim a partial credit. The calculation is ($175,000 − $160,000) ÷ $20,000 = 75%, which means they are not able to claim 75 percent of the credit. Since they have incurred more than $4,000 of qualifying expenses, they would normally be able to claim the maximum $2,500 credit, but now they are only able to claim $625 for the American Opportunity tax credit. $2,500 - ($2,500 × .75) = $625

Brendan is covered by a group term life insurance policy that provides coverage equal to three and a half times his annual salary of $100,000 with a cap of $325,000 of maximum coverage. Brendan's employer pays the entire cost of the policy, for which the uniform premium is $0.66 per $1,000 of coverage per month. How much of this premium is taxable to Brendan, assuming he is covered for 9 months? A) $2,178 B) $1,634 C) $2,500 D) $182

The correct answer is (B). The cost of up to $50,000 of coverage is nontaxable. $325,000 − 50,000 = $275,000 [($275,000 ÷ $1,000) × $0.66] × 9 months = $1,634 taxable coverage

Conner has a real estate asset used in his business. He exchanges it for a like-kind real estate asset owned by Darby. (Conner and Darby are not related.) The basis of Conner's asset is $50,000. He gives Darby $10,000 cash plus the asset in exchange for Darby's asset, which is worth $46,000. Darby's basis in his original asset is $10,000. What is Darby's recognized gain or loss? A) $0 B) $10,000 C) $26,000 D) $36,000

The correct answer is (B). The values exchanged must be equal. Conner gives his property plus $10,000 in exchange for a property worth $46,000 therefore Conner's property is valued at $36,000.

Marcus bought a house 10 months ago. His job recently transferred him to an office in a different state, so he sold his house. If he originally purchased the home for $300,000 and sold it for $600,000, what amount of gain from the sale of the personal residence can he exclude from gross income? Assume he uses the single tax-filing status. A) $300,000 B) $104,167 C) $250,000 D) $145,833

The correct answer is (B). Although he did not live in the house for a full 2 years, Marcus is eligible for a prorated exclusion because of his change in employment. Therefore he is eligible for a maximum exclusion of $104,167, or (10 ÷ 24) × $250,000.

In the first year, Eli invested $100,000 for a 20 percent partnership interest in an activity in which he is a material participant. The partnership reported a loss of $400,000 in the first year and $200,000 in the second year. Eli's share of the partnership's loss was $80,000 in the first year and $40,000 in the second year. How much of the loss from the partnership can Eli deduct? A) First Year: $80,000; Second Year: $40,000 B) First Year: $80,000; Second Year: $20,000 C) First Year: $0; Second Year: $0 D) First Year: $80,000; Second Year: $0

The correct answer is (B). Eli's losses are deductible in both years because he is a material participant in the activity. However, the at-risk rules limit his total losses to $100,000.

Cara spent $15,000 for day-care services for her four children so that she could go to work. If her adjusted gross income (AGI) is $180,000, how much is her dependent-care credit? (Please ignore changes from the American Rescue Plan Act of 2021.) A) $0 B) $1,200 C) $3,000 D) $15,000

The correct answer is (B). For taxpayers with AGI over $43,000, the dependent-care credit provides a credit of 20 percent of the cost up to $3,000 per qualifying child or $6,000 for two or more children. Therefore her credit is $1,200, that is, $6,000 × 0.20.

Sid bought 10 shares of Lazy Dog, Inc., stock on January 1st of the current year. Sid paid $20 for each share. At first, it appeared that Sid had made a good investment as the price of Lazy Dog stock rose to $50 per share on March 1st . However, rumors of corporate wrongdoing soon started to circulate, and the price of Lazy Dog began to fall. On August 1st, Lazy Dog, Inc., declared bankruptcy and announced that the stockholders should not expect to receive any of the proceeds from the liquidation of the corporation. What type of loss, if any, does Sid have in the current year? A) Sid does not have a loss because he did not sell the stock. B) He has a short-term capital loss of $200. C) He has a short-term capital loss of $500. D) He has a long-term capital loss of $200.

The correct answer is (B). IRC Section 165 creates an artificial sale date of December 31 for worthless securities. Sid is deemed to have sold the stock for $0 on December 31. Because Sid's holding period was less than one year and one day, his loss will be a short-term capital loss. The amount of Sid's loss is equal to his basis in the investment.

Ryann purchased 10,000 shares of SDN, Inc., 5 years ago for $100,000. She recently gave those shares to her son, Jacob. The value of the 10,000 shares of stock as of the date of the gift was $60,000. Which of the following statements is true? A) If Jacob subsequently sells the shares of SDN, Inc., for $105,000, the basis used to calculate his gain or loss will be $60,000. B) If Jacob subsequently sells the shares of SDN, Inc., for $40,000, the basis used to calculate his gain or loss will be $60,000. C) If Jacob subsequently sells the shares of SDN, Inc., for $40,000, the basis used to calculate his gain or loss will be $100,000. D) If Jacob subsequently sells the shares of SDN, Inc., for $105,000, he will not have any gain or loss.

The correct answer is (B). Jacob will have a double basis in the stock, determined as follows: If the donee's sale price is between the donor's original basis and the fair market value (FMV), then there is no gain or loss. If Jacob subsequently sells the shares of SDN, Inc., for $40,000 (or less than the FMV on the date of the gift), the basis used to calculate his gain or loss will be $60,000. If he subsequently sells the shares for a price higher than the donor's basis on the date of the gift ($100,000), the basis used to calculate his gain is $100,000.

Lauren paid $3,000 in interest on her student loans this year. Lauren is single and earned $69,000 from her job as an analyst and $8,500 in interest income. She had no other income this year. What amount of Lauren's student loan interest is deductible for adjusted gross income (AGI) this year? A) $0 B) $1,250 C) $1,500 D) $2,500

The correct answer is (B). Lauren's AGI of $77,500 is within the phase-out range, so the maximum deduction of $2,500 will be reduced. The deductible amount is [($85,000 maximum in range − $77,500 AGI) ÷ $15,000 range] × $2,500 maximum deduction = $1,250.

Which of the following can be excluded from Virginia's gross income? I. The value of a diamond ring that Virginia received as a gift from Graham II. The value of a mansion that Virginia inherited from her parents III. The value of concert tickets that Virginia won in a radio contest IV. The value of a scholarship for room and board that Virginia received to her state university A) I only B) I and II C) I, II, and III D) II, III, and IV

The correct answer is (B). Only the gifts can be excluded from Virginia's gross income. The concert tickets that Virginia won in a radio contest are a prize/award and therefore the value is included in her gross income. The value of the scholarship is also included in Virginia's gross income because the scholarship was for room and board, which is not considered to be qualified tuition and related expenses.

What is an advantage of using the Section 179 deduction over the Modified Accelerated Cost Recovery System (MACRS)? A) By deducting more currently, the total tax liability is increased and the present value of cash flows is decreased. B) The Section 179 may be elected on a property-by-property basis. C) Section 179 reduces the depreciation on most assets to only 5 years. D) Section 179 applies only to business assets, whereas depreciation applies to both business assets and personal assets.

The correct answer is (B). Section 179 is an upfront business deduction that can be elected on a property-by-property basis, whereas MACRS 100 percent bonus depreciation applies to the entire class of property, and an election to opt out applies to the entire class of property. Section 179 can be used by businesses to reduce current tax liabilities. By deducting more currently, total tax liability is reduced, and the present value of cash flows is increased.

Which of the following fringe benefits received by an employee would be excluded from the employee's gross income? Employer-provided parking Dues to an athletic club paid for by the employer Tickets to the basketball game tonight Employer-provided interior decorating for a new personal residence A) I and II B) I and III C) II and IV D) I, III, and IV

The correct answer is (B). Statement I would be excluded as a qualified transportation fringe benefit. Statement II would be included because club dues paid by the employer must be included in the employee's gross income. The athletic facility must be on premises owned or leased by the employer to be excluded from the employee's gross income. Statement III is excluded from gross income because occasional tickets to a sporting event would not be included in gross income. However, season tickets would be included in the employee's gross income. Statement IV would be included in gross income because it is a personal expense paid for by the employer.

Mattie, a single mother, has four children, aged 6, 7, 9, and 17. How much will her Child Tax Credit be for the current tax year assuming she is under the adjusted gross income (AGI) threshold? (Please ignore changes from the American Rescue Plan Act of 2021.) A) $1,000 B) $6,000 C) $6,500 D) $8,000

The correct answer is (B). The 17-year-old is not considered a qualifying child for the purposes of the Child tax credit. Mattie will receive $6,000, or a $2,000 credit for each of her other three children. The 17 year-is considered an "Other Dependent" who will allow Mattie to claim a $500 Family Credit.

Mohammed has decided to make charitable contributions of property this year. He donates a piece of art that he created to a local art museum. The artwork has an adjusted basis of $900 and a fair market value of $40,000. His adjusted gross income (AGI) is $90,000. What is his charitable deduction for this year? A) $0 B) $900 C) $36,000 D) $40,000

The correct answer is (B). The deduction for a work of art created by the donor is limited to the creator's adjusted basis in the property.

Claire gifts her brother, Max, stock in EDD Company. The value of the stock on the date of the gift was $14,000. Claire had acquired the stock for $17,000 5 years ago. What is the tax consequence to Max if Max sells the stock for $12,000 one month after the gift? A) A short-term capital gain B) A short-term capital loss C) A long-term capital loss D) No gain or loss

The correct answer is (B). The holding period begins at the date of the gift when the loss basis is used as a result of the double-basis rule. The loss is $2,000 and is a short-term capital loss.

Sebastian is starting a new business. He is concerned about liability. He would like to have flow-through taxation. At some point, he would like to be able to easily sell interests in the business, but he does not expect to have more than 20 investors. He does not want to pay self-employment taxes on all income. Which of the following entities would best suit his needs? A) A proprietorship B) An S corporation C) A C corporation D) A partnership

The correct answer is (B).Option (A) is not correct because a proprietorship does not provide limited liability and he would need to change entities to take on future investors. Option (C) is not correct because a C corporation would not provide flow-through taxation. Option (D) is incorrect because a partnership would not provide limited liability and he would need to pay self-employment taxes on the business net income. Only option (B) meets all of Sebastian's requirements.

All of the following statements concerning the alternative minimum tax (AMT) as it applies to individual taxpayers are correct EXCEPT: A) Some itemized deductions available for regular tax purposes must be added back to regular income to determine income under the AMT. B) Taxpayers are permitted to take the standard deduction for both regular tax purposes and AMT purposes. C) Adjustments made to itemized deductions when calculating AMT result in a permanent increase in tax. D) Charitable deductions are claimed in the same way for regular income tax purposes as for AMT purposes.

The correct answer is (B).Taxpayers who do not itemize deductions take the standard deduction for regular tax purposes, but this is added back to alternative minimum taxable income for AMT purposes. All of the other statements are correct.

Melanie had to pay alternative minimum tax (AMT) last year. She had to add several items to her regular taxable income in arriving at her alternative minimum taxable income (AMTI). Which of the following items results in an AMT credit that can be used to offset future regular tax liability? A) $10,000 in property taxes paid on her principal residence if paid in advance B) A $100,000 difference between the fair market value (FMV) of stock and the strike price in the incentive stock option (ISO) used to purchase the stock C) $5,000 in interest on private activity municipal bonds D) $7,000 in additional medical expenses

The correct answer is (B).The inclusion of the difference between the FMV and exercise price of the stock options will result in a credit that Melanie can use against future regular income tax liability. Options (A) and (C) are adjustments that result in permanent differences in tax liability as a result of the imposition of the AMT. Medical expenses are deductible to the same extent under both regular tax and AMT.

Savannah owns an event planning company. Several years ago, she purchased a magnificent fondue fountain for $3,000 and has since taken $1,200 in depreciation deductions on the fountain. She is now ready to replace the fountain with tools for creating ice sculptures, but she is not sure what the tax consequence of selling the fountain will be. Which of the following statements is true regarding the tax consequence of selling the fountain? A) If she sells the fountain for $1,800, she will have a $1,200 ordinary loss. B) If she sells the fountain for $1,700, she will have a $100 capital loss. C) If she sells the fountain for $2,000, she will have an ordinary gain of $200 and no capital gain. D) If she sells the fountain for $3,300, she will have a $1,500 capital gain.

The correct answer is (C). Option A Option B Option C Option D Amount Realized $1,800 $1,700 $2,000 $3,300 Adjusted Basis $1,800 $1,800 $1,800 $1,800 Gain/Loss $0 -$100 $200 $1,500 Tax Impact None Ordinary Loss Ordinary Gain Part Ordinary Gain ($1,200)/Part Capital Gain ($300)

Amon owns a personal fitness facility that specializes in training athletes. Several years ago, he purchased a treadmill for $5,000 and has since taken $1,000 in depreciation deductions. He is now ready to replace the treadmill with a more current model, but he is not sure what the tax consequences of selling the old treadmill will be. Which of the following statements regarding the tax consequences of selling the old machine is true? A) If Amon sells the old treadmill for $4,000, he will have a $600 capital loss. B) If Amon sells the old treadmill for $3,800, he will have $600 in ordinary income. C) If Amon sells the old treadmill for $4,200, he will have $200 in ordinary income. D) If Amon sells the old treadmill for $4,800, he will have an $800 capital gain.

The correct answer is (C). $5,000 $4,200 − $1,000 − $4,000 $4,000 (Basis) $200 (Ordinary Income)

Shana exchanged Asset 1 (realty) with Joshua in return for Asset 2 (realty) and $100,000. Asset 1 and Asset 2 are like-kind assets (real estate). Shana's basis in Asset 1 is $80,000. Joshua's Asset 2 has a fair market value (FMV) of $300,000 and a basis of $320,000. What is Shana's recognized gain or loss on the exchange? A) $0 B) $80,000 C) $100,000 D) $320,000

The correct answer is (C). Shana must recognize $100,000 as gain (the boot received). The values exchanged must be equal. Shana gives her property to Joshua in exchange for his $300,000 property plus $100,000 therefore Shana's property is valued at $400,000.

On January 1, Elena loaned her son, Juan, $90,000 to purchase a new personal residence. There were no other loans outstanding between Elena and Juan. Juan's only income was a $30,000 salary and $4,000 in interest income. Elena had investment income of $200,000. Elena did not charge Juan interest. The relevant AFR was 2 percent. For the current year A) Juan must recognize imputed interest income of $1,800. B) Elena must recognize imputed interest income of $4,000. C) Elena must recognize imputed interest income of $1,800. D) Juan is allowed a deduction for imputed interest of $4,000.

The correct answer is (C). Since the loan is not more than $100,000, the imputed interest will be the applicable federal rate minus the actual rate, limited to Juan's net investment income. The imputed interest would be $1,800, that is, 2% × $90,000. Elena would impute the lesser of the imputed interest using the AFR and Juan's net investment income. Since the interest imputed using the AFR is less than Juan's net investment income, only $1,800 is recognized by Elena.

Harold is covered by a $170,000 group term life insurance policy, and his daughter is the beneficiary. Harold's employer pays the entire cost of the policy, for which the uniform premium is $1.27 per $1,000 of coverage per month. How much of this premium is taxable to Harold, assuming he is covered for 12 months? A) $0 B) $152 C) $1,829 D) $2,591

The correct answer is (C). The cost of up to $50,000 of coverage is nontaxable. $170,000 − 50,000 = $120,000 [($120,000 ÷ $1,000) × $1.27] x 12 months = $1,829 taxable coverage

Abigail completed her freshman year of college. Her parents paid $8,000 in qualified education expenses this year. Abigail's parents are married-filing-jointly (MFJ) taxpayers and had an adjusted gross income (AGI) of $200,000 for the current tax year. What, if any, education credit will provide them with the highest credit, and how much is that credit? A) Abigail's parents can claim a Lifetime Learning credit in the amount of $8,000. B) Abigail's parents can claim a Lifetime Learning credit in the amount of $1,600. C) They are not eligible to claim any education tax credits since their AGI exceeds the phaseout. D) Abigail's parents can claim an American Opportunity tax credit in the amount of $2,500.

The correct answer is (C). The modified adjusted gross income (MAGI) phaseout for both the American Opportunity tax credit and Lifetime Learning credit is $160,000 to $180,000 for MFJ taxpayers.

Lauren and Dylan got married and bought a house 13 months ago. Lauren's employer recently transferred her to an office in a different state, so Lauren and Dylan sold their residence. What is the maximum amount of gain that Lauren and Dylan can exempt from income taxation? A) $0 B) $229,167 C) $270,833 D) $500,000

The correct answer is (C). Although they did not live in their house for a full 2 years, Lauren and Dylan are eligible for a prorated exclusion because of Lauren's change in employment. Therefore they are eligible for a maximum exclusion of $270,833, that is, (13 ÷ 24) × $500,000.

Which of the following creates a potential Section 1245 result? A) The amortization of goodwill B) The immediate expensing under Section 179 C) The sale of tangible personalty used in a trade or business at a gain D) A Section 1231 loss from a property's disposition

The correct answer is (C). Depreciable property includes equipment, patents, copyrights, and other intangibles. Option (A) is incorrect because goodwill is amortized. Option (B) is incorrect because Section 1245 is not applied to Section 179 expensing. Option (D) is incorrect because Section 1231 is applied to gains recognized after recapture.

With regard to the alimony deduction related to a post-1984, pre-2019 divorce, which one of the following statements is correct? A) Alimony is deductible by the payer-spouse and includible by the payee-spouse to the extent that payment is contingent on the status of the divorced couple's children. B) The divorced couple may be members of the same household at the time alimony is paid, provided that the persons do not live as a married couple. C) Alimony payments must terminate on the death of the payee-spouse. D) Alimony may be paid either in cash or in property.

The correct answer is (C). For post-1984, pre-2019 divorce agreements and decrees, payments to former spouses are alimony only if the payments are in cash. the agreement or decree does not specify that the payments are not alimony for federal income tax purposes. the payee and payer are not members of the same household at the time the payments are made. there is no liability to make the payments for any period after the death of the payee.

Which of the following statements is correct with regard to the concepts of realization and recognition? A) All gains that are realized must be recognized in the same tax year in which they are realized. B) All losses that are realized must be recognized in the same tax year in which they are realized. C) In order for gains to be recognized, they need to have been realized. D) Losses of any kind are never allowed to be realized or recognized.

The correct answer is (C). Generally speaking, in order for income to be recognized, a transaction must have occurred to realize the income. There are tax-deferral mechanisms in place that allow gains to be recognized in different years than the actual gain was realized or possibly never recognized at all (exclusion of gain on sale of principal residence, for example).

Last year, Sam had a Section 179 deduction carryover of $8,000. In the current year, he elected Section 179 for an asset acquired at a cost of $10,000. Sam's net income for the current year is $15,000. Determine Sam's Section 179 deduction for the current year. (Assume no other income.) A) $8,000 B) $10,000 C) $15,000 D) $18,000

The correct answer is (C). His Section 179 deduction is limited to $15,000 (due to the net income limitation). A Section 179 deduction cannot create a loss. He would still have a $3,000 Section 179 carryover from the previous year.

Hank is covered by a group term life insurance policy that provides coverage of $225,000. Hank's employer pays the entire cost of the policy, for which the uniform premium is $0.23 per $1,000 of coverage per month. How much of this premium is taxable to Hank, assuming he is covered for 12 months? A) $828 B) $69 C) $40 D) $483

The correct answer is (D). The cost of up to $50,000 of coverage is nontaxable.$225,000 − $50,000 = $175,000[($175,000 ÷ $1,000) × $0.23] × 12 months = $483 taxable coverage

Ali and Dana are married and are trying to calculate their gross income for the current year. Which of the following items should they include in gross income? I. Child support payments in the amount of $15,000 received from Dana's ex-husband for the support of their minor childII. $1,200 in dividends receivedIII. Unemployment benefits received in the amount of $800IV. $3,000 that Dana earned selling homemade soaps A) IV only B) I and II C) II, III, and IV D) All of the above

The correct answer is (C). Option I is not correct because child support is not includible in gross income. Dividend income, unemployment compensation benefits, and gross income from self-employment are included in gross income.

Antonio has taxable income of $350,000 and is concerned about being subjected to the alternative minimum tax (AMT). The following items were included in calculating his taxable income. Which one of the following may be deductible from regular taxable income in calculating AMT? A) He had charitable deductions of $50,000. B) He had a casualty loss of $25,000. C) He had property tax deductions of $10,000. D) He had qualified housing interest of $30,000.

The correct answer is (C). Property, sales, use, and state income taxes are deductible for regular income taxation purposes, but they are not deductible for AMT purposes. The other items are treated in the same way for both regular tax purposes and AMT purposes.

All of the following are deductions for adjusted gross income (AGI) EXCEPT: A) maintenance expenses for a rental property actively managed by the taxpayer. B) moving expenses of a taxpayer who is an active duty member of the Armed Forces. C) real estate taxes. D) one-half of self-employment tax paid.

The correct answer is (C). Real estate taxes are deductible from AGI and are limited to $10,000 after 2017. Moving expenses are not deductible for most people, but they are deductible for AGI for members of the Armed Forces.

Rose pursued a hobby of selling used computers in her spare time. During the year she sold the computers for $3,000. She incurred expenses as follows: $2,000 for the cost of goods sold $1,200 for supplies $800 in interest on a loan to start a business $750 in advertising fees Assuming that the activity is a hobby, and that she cannot itemize this year, how should she report these items on her tax return? A) Include $3,000 in income and deduct $4,750 for adjusted gross income (AGI). B) Ignore both income and expenses since hobby losses are disallowed. C) Include $3,000 in income and deduct nothing for AGI. D) Include $3,000 in income and deduct interest of $800 for AGI.

The correct answer is (C). Rose must include the $3,000 in income and deduct nothing. Hobby expenses are not deductible after 2017.

Victor and Charlie own a house at the beach. The house was rented to unrelated parties for 8 full weeks during the current year. Victor and Charlie used the house for 16 days for their vacation during the year. After properly dividing the expenses between rental and personal use, it was determined that a loss was incurred as follows: Gross rental income $6,400 LessMortgage interest and property taxes$7,000 Other allocated expenses$1,000− 8,000 Net rental loss - $1,600 What is the correct treatment of the rental income and expenses on Victor and Charlie's joint income tax return for the current year? A) A $1,600 loss should be reported. B) The rental portion of interest and taxes can be deducted. C) The rental expenses (other than interest and taxes) are limited to the gross rental income in excess of deductions for interest and taxes allocated to the rental use. D) Since the house was used only 20 percent personally by Victor and Charlie, all expenses allocated to personal use may be deducted.

The correct answer is (C). Since the amount of time the property was used for personal reasons (16 days) was more than the greater of 14 days and 10 percent of the time the property was rented out (5.6 days), the property will not qualify as primarily rental property. The property will be classified as mixed-use property. Therefore no loss is allowed.

Carmen, an unmarried taxpayer using the single filing status, received $25,000 of Social Security benefits this year. Carmen also received $2,000 of interest income and $45,000 of income from her retirement plan during the year. How much of Carmen's Social Security must be included in her gross income this year? A) $25,000 B) $12,500 C) $21,250 D) $0

The correct answer is (C). Since the taxable income is over the base limit, 85 percent of the $25,000 of Social Security benefits must be included in Carmen's gross income this year. ($2,000 + $45,000) + (0.50 × $25,000) = $59,500 > $25,000 0.85 × $25,000 = $21,250

Phaedra and Rebekah, both aged 48, are married and filed a joint federal income tax return for the current year. Their adjusted gross income (AGI) was $113,000, $95,000 of which was Phaedra's salary. Rebekah had no income of her own. Neither spouse was covered by an employer-sponsored retirement plan. What amount could they contribute to IRAs this year to take advantage of their maximum allowable IRA deduction for their return? A) $0 B) $6,000 C) $12,000 D) $14,000

The correct answer is (C). Since they are not covered by a retirement plan, they are allowed a deductible $6,000 IRA contribution for Phaedra (2022) plus a $6,000 deductible contribution to a spousal IRA (2022). A total deductible IRA contribution of $12,000 is allowed.

During the current year, Silas, a self-employed individual, paid the following amounts: $5,000 in federal income tax $3,000 in state income tax $800 in real estate taxes on land (held as an investment) $600 in state sales taxes A $400 state occupational license fee What amount can Silas deduct as taxes by itemizing his deductions? A) $3,000 B) $3,400 C) $3,800 D) $4,800

The correct answer is (C). State income and real estate taxes are deductible as itemized deductions. Federal income taxes paid are deductible from the tax liability but are not itemized deductions. A taxpayer may deduct the greater of the state income tax and state sales tax, but he may not deduct both. Occupational license fees are deductible as a direct business cost, but they are deducted from adjusted gross income (AGI), not as itemized deductions.

How much in the form of Social Security and Medicare taxes must a taxpayer who is an employee earning $200,000 pay in 2022? A) $11,754 B) $15,300 C) $12,014 D) $12,550

The correct answer is (C). The Social Security wage base for 2022 is $147,000, and it's taxed at 6.2 percent.6.2% × $147,000 = $9,114 There is no wage base for Medicare FICA taxes, his gross pay of $200,000 is taxed at 1.45 percent to the employee.1.45% × $200,000 = $2,900 $9114 + $2,900 = $12,014

Margaret died owning a passive investment in which her basis was $25,000. The fair market value (FMV) of the asset included in her estate was $150,000, and she had $200,000 of suspended passive-activity losses associated with the asset at the time of her death. Which of the following is the amount of suspended losses that can be deducted on Margaret's final tax return? A) $0 B) $25,000 C) $75,000 D) $200,000

The correct answer is (C). The asset will receive a step up in basis to $150,000, a $125,000 increase. The suspended losses are reduced by the amount of the step up, leaving $75,000, or $200,000 suspended losses − $125,000 basis step up, that can be deducted on her final tax return.

Kristen is single, and her share of qualified business income (QBI) from a partnership is $135,000. Her adjusted gross income (AGI) is $150,000. Her total taxable income from all sources before taking the 20 percent deduction for QBI is $125,000. What is the amount of the Section 199A QBI deduction that Kristen can take this year? A) $30,000 B) $27,000 C) $25,000 D) $20,000

The correct answer is (C). The deduction is the lesser of 20 percent of Kristen's QBI and 20 percent of taxable income. Twenty percent of her QBI is $27,000. Twenty percent of her taxable income is $25,000. Her deduction is $25,000.

Molly invested $100,000 in an annuity contract. Years later, she annuitized the contract. The insurance company agreed to pay her $1,666.67 per month for 20 years. How much of each payment is taxable? A) $0 B) $466.67 C) $1,250.00 D) $1,666.67

The correct answer is (C). The exclusion ratio can be determined by dividing Molly's investment in the contract by her expected total return. $100,000 ÷ $400,000 = 25% exclusion ratio 1 − 0.25 = 0.75 inclusion × $1,666.67 = $1,250

Carl would like to invest in bonds and is considering either a corporate bond with an interest rate of 6 percent or a tax-exempt municipal bond of comparable risk and quality. Carl's marginal tax rate is 22 percent. In order to help Carl compare these two bonds, what interest rate would the municipal bond need to earn for the tax-equivalent yield to be equal to that of the corporate bond? A) 5.00% B) 4.50% C) 4.68% D) 6.00%

The correct answer is (C). The tax-equivalent yield for a municipal bond with an interest rate of 4.68 percent would be 6 percent or (0.06 x (1 - 0.22).

Which of the following are characteristics of a C corporation? I. The number of shareholders is limited.II. The bankruptcy of a major shareholder has no effect on the business form.III. Shareholder liability is limited.IV. The capital structure is dependent upon the resources of the initial shareholders. A) I and II B) III and IV C) II and III D) I, II, III, and IV

The correct answer is (C).Statements II and III are accurate. The bankruptcy of a major shareholder has no effect on the business form. The number of shareholders in a corporation is unlimited. Capital may be raised by the sale of stock and by borrowing in capital markets and is not limited to the resources of the initial shareholders.

All of the following statements regarding family limited partnerships (FLPs) are correct EXCEPT: A) The primary purpose of an FLP is to transfer assets to younger generations of a family by using annual exclusion gifts and valuation discounts. B) Upon the creation of an FLP, there are neither income nor gift tax consequences because the entity created is owned by the same person(s) who owned it prior to the transfer. C) Publicly traded securities make an ideal asset to transfer to an FLP. D) The use of an FLP can help protect family assets.

The correct answer is (C).The creation of family limited partnerships and the use of discounts to transfer value at a lower gift tax cost has been regularly contested by the IRS. The risk of the IRS contesting any discounts for publicly traded securities is a significant risk associated with the FLP.

Rowan owns a downtown office building. She originally purchased the building for $2,000,000 and took straight-line depreciation deductions of $1,200,000. Rowan is in the 37 percent tax bracket. What are the tax consequences if Rowan sells the building for $2,100,000? (Ignore the Medicare tax on net investment income.) A) Rowan will have ordinary income of $0. B) Rowan will have $1,200,000 of gain taxed at 25 percent. C) Rowan will have Section 1231 gains of $100,000 taxed at 20 percent. D) Rowan will have all of the above.

The correct answer is (D). $2,100,000 Sale Price − 800,000 Adjusted Basis $1,300,000 Gain (Total) $1,200,000 at 25% 100,000 at 20% The capital rate in this case will be 20 percent because Rowan is in the 37 percent tax bracket.

Brianna owns a downtown office building. She originally purchased the building for $600,000 and deducted straight-line depreciation deductions of $200,000. What are the tax consequences if she sells the building for $1,000,000? A) She will have ordinary income of $0. B) She will have $200,000 of any gain taxed at 25 percent. C) She will have Section 1231 gains of $400,000. D) She will experience all of the above.

The correct answer is (D). Sale Price $1,000,000 Less: Adjusted Basis − $400,000 = ($600,000 − $200,000) Gain $600,000 Breakdown of Gains: $200,000 is unrecaptured Section 1250 gain (straight-line depreciation). This is net capital gain taxed at 25 percent. $400,000 is net capital gain taxed at the regular capital-gains rates. $1,000,000 minus $600,000 equals $400,000 taxed at capital-gains rates.

Seth receives incentive stock options (ISOs) with an exercise price of $32 when the stock is trading at $32. Seth exercises these options 2 years after the date of the grant, when the stock price is $50 per share. Which of the following statements is correct? A) Seth will have W-2 income of $32 per share upon exercise. B) Seth will have W-2 income of $50 per share upon exercise. C) Seth will have $32 of alternative minimum taxable income (AMTI) upon exercise. D) Seth's adjusted basis for regular income tax purposes will be $32 at exercise

The correct answer is (D). Seth does not have regular income at the date of exercise. Seth's adjusted basis will be $32. The AMT adjustment will be the difference between the fair market value and the exercise price. $50 − $32 = $18

Ellen bought a house 6 months ago. Her job recently transferred her to an office in a different state, so she sold her house. If she originally purchased the home for $500,000 and sold it for $750,000, what amount of gain from the sale of the personal residence must she include in gross income? Assume she uses the single tax-filing status. A) $0 B) $62,500 C) $250,000 D) $187,500

The correct answer is (D). Although she did not live in the house for a full 2 years, Ellen is eligible for a prorated exclusion because of her change in employment. Therefore she is eligible for a maximum exclusion of $62,500, or (6 ÷ 24) × $250,000. $250,000 gain − $62,500 exclusion = $187,500.

Which of the following is (are) deductible for adjusted gross income (AGI)? I. Alimony paid to the taxpayer's ex-spouse (2016 divorce) II. Capital losses III. Ordinary and necessary expenses incurred in a business IV. A contribution to a Roth IRA V. Child support paid to the taxpayer's ex-spouse A) III and IV B) I, III, and V C) II, III, and V D) I, II, and III

The correct answer is (D). Child support payments are excluded from income for the parent receiving the support, and the parent paying the support does not get a deduction. Contributions to Roth IRAs are not deductible for AGI.

All of the following qualifies for the exception to the 10 percent early withdrawal penalty for IRAs EXCEPT: A) Distribution at death B) Distributions after reaching the age of 59½ C) Distributions due to disability D) Distributions of Traditional IRA contributions

The correct answer is (D). Distributions of Roth IRA contributions avoid the 10 percent early withdrawal penalty. A distribution of a Traditional IRA contribution will result in a 10 percent penalty as well as an income tax liability.

Which of the following is true regarding health savings accounts (HSAs)? A) HSAs do not have an annual funding limit, while FSAs do have such a limit. B) An HSA's only benefit is the tax-deferral of savings placed in the accounts, and taxpayers do not receive a deduction for contributions made to the account. C) HSA distributions used for nonqualifying expenses are not subject to any additional penalty. D) HSA contributions can be made by both the employee and the employer up to a certain annual limit.

The correct answer is (D). HSA contributions can be made by both the employee and the employer up to a certain annual limit. HSAs have annual funding limits, taxpayers receive an income tax deduction for contributions made, and distributions used for nonqualifying expenses are subject to a 20 percent penalty.

Which of the following statements about the alternative minimum tax (AMT) is correct? A) A taxpayer may elect to pay tax based on the AMT calculation if doing so produces a lower tax result. B) The AMT is designed primarily to change the timing of tax payments so that there will be more tax payments in the future. C) The AMT encourages efforts of taxpayers to participate in activities that reduce or eliminate their current tax liability. D) Some adjustments made for AMT purposes result in a permanent increase in tax.

The correct answer is (D). If the calculated tax due is greater under the AMT, the taxpayer must pay the higher amount. The AMT is not a voluntary alternative to the regular tax system; it is a mandatory alternative and applies only when the AMT exceeds the regular tax imposed on the taxpayer. The AMT is designed primarily to change the timing of tax payments so that there are more current tax payments. The AMT frustrates efforts of taxpayers to participate in activities that reduce or eliminate their current tax liability.

Marcia and Darius are married, and together they have an adjusted gross income (AGI) of $60,000. They have no dependents, and they file a joint federal income tax return. Each pays $4,800 for medical insurance. During the year, they paid the following amounts for medical care, for which they were not reimbursed by insurance: $3,600 in doctor and dentist bills and hospital expenses $800 in prescription drugs Determine the deduction allowable for medical expenses paid during the current year. A) $14,000 B) $3,600 C) $4,500 D) $9,500

The correct answer is (D). Marcia and Darius can claim a medical expense deduction of $9,500, or $14,000 − $4,500, for the current year. The total medical expenses paid equals $14,000, or $4,800 + $4,800 + $3,600 + $800, and the expenses above the 7.5 percent AGI limitation of $4,500, or 7.5% × $60,000, are deductible. $14,000 minus $4,500 equals the allowable $9,500 medical expense deduction.

Omar, a professor of literature, earned a salary of $140,000 from a university in the current year. He received $35,000 in dividends and interest during the year. In addition, he incurred a loss of $25,000 from an investment in a passive activity. His at-risk amount in the activity at the beginning of the current year was $15,000. What is Omar's adjusted gross income (AGI) for the current year? A) $115,000 B) $150,000 C) $160,000 D) $175,000

The correct answer is (D). Omar's AGI, after considering the passive investment, is $175,000, that is, $140,000 active income + $35,000 portfolio income. He would not be allowed to offset the passive loss against active or portfolio income.

Layla and Warren purchased a vacation home jointly 5 years ago for $400,000. Now, Layla and Warren are getting divorced. As part of the divorce settlement, Layla receives the vacation home, which is worth $680,000. Which of the following statements is true? A) If Layla sells the vacation home 6 months after receiving it in the divorce settlement, any gain or loss that she has will be short-term. B) If Layla sells the vacation home for $800,000, she will have a $120,000 gain. C) In any future sale of the vacation home, Layla and Warren will each have a basis of $200,000. D) If Layla sells the vacation home for $780,000, she will have a gain of $380,000.

The correct answer is (D). Option (A) is incorrect because Layla has a carryover holding period, so if she sells the vacation home 6 months after receiving it in the divorce settlement, any gain or loss that she has will be long-term. Option (B) is incorrect because Layla has a carryover basis in the property. Therefore her basis is $400,000. Option (C) is incorrect because Warren no longer has an interest in the property.

Section 1245 recapture does not apply to business equipment held for 17 months or longer if A) the property was destroyed by fire and the insurance recovery exceeds the property's adjusted basis. B) the property was sold for a gain but was depreciated using straight-line depreciation rather than MACRS. C) the property was acquired, depreciated, and exchanged for a less valuable asset and the buyer of the asset paid additional cash in the exchange. D) the property was abandoned as worthless.

The correct answer is (D). Option (A) is incorrect because if the insurance proceeds exceed the property's adjusted basis, the excess is considered a sale and any portion of gain attribution to depreciation will be subject to Section 1245 recapture. Option (B) is incorrect because Section 1245 recapture occurs any time a gain results from the reduction of basis due to depreciation. Option (C) is incorrect because Section 1245 recapture carries over to the replacement property in nontaxable exchanges. Option (D) is correct. Property sold or abandoned below the basis adjusted by depreciation is not subject to Section 1245 recapture because not all depreciation was taken or there was more likely a loss than a gain. For Section 1245 recapture to occur, there must be a gain over the basis.

Richard broke his leg skiing earlier this year. He collected $12,000 from his short-term disability plan, for which his employer paid 80 percent of the premiums. How much must Richard include in his gross income? A) $0 B) $2,400 C) $12,000 D) $9,600

The correct answer is (D). Richard must include 80 percent of the disability benefits since his employer paid 80 percent of the premiums.

Dean dies, leaving Jessica stock of OK Company. Dean had acquired the stock in November of this year and died December 10th of this year. The value of the stock on the date of death was $1,000, and Dean's adjusted taxable basis was $1,200. Presuming that Jessica sells the stock for $1,050 on February 14th of the next year, what is her tax consequence? A) She is subject to the double-basis rule: no gain or loss. B) She has a short-term capital loss of $150. C) She has a long-term capital loss of $150. D) She has a long-term capital gain of $50.

The correct answer is (D). The shares take on the fair market value (FMV) of $1,000 at Dean's death. Dean's basis is irrelevant. All inherited assets are long-term regardless of the holding period as death is presumed involuntary.

Glen and Sandra are married-filing-jointly taxpayers with three children, aged 6, 12, and 16. Their MAGI is $68,500. What is the amount of the Child Tax Credit that they can claim for the current tax year? (Please ignore changes from the American Rescue Plan Act of 2021.) A) $2,000 B) $3,000 C) $5,000 D) $6,000

The correct answer is (D). They have three children. The Child tax credit is $2,000 per child. To be eligible, the child must be under the age of 17.

Which of the following is not a disallowed loss? A) Losses from wash sales B) Losses from the sale of personal use assets C) Losses from the subsequent sale of property gifted to a related party when its fair market value is less than the original owner's adjusted basis and the sale is greater than the loss basis but less than the gain basis of the transferor D) Capital losses in excess of $3,000

The correct answer is (D). Up to $3,000 of capital losses may be recognized against sources of income other than capital gains in any one tax year. However, if a taxpayer has capital losses in excess of $3,000, these losses are not disallowed but are carried forward indefinitely to future tax years.

Nadia and Callum were married, but Callum recently died. Callum paid $200,000 for their home 5 years ago. Its fair market value (FMV) was $300,000 when he died. What is Nadia's basis in the home after Callum's death if the home was held as community property and Callum left his half to Nadia? A) $150,000 B) $200,000 C) $250,000 D) $300,000

The correct answer is (D). Upon the death of either spouse in a community-property state, both halves of community property are stepped to the fair market value regardless of who inherits the decedent's half.

Jerry is a 20 percent partner in a partnership that produced business income (before adjustments) of $350,000 this year, and he is a 70 percent owner in an S corporation that produced net business income of $50,000 this year. The total amount of self-employment taxes allocated to his net business income is $4,945. His total taxable income is $101,000. What are Jerry's qualified business income (QBI) and QBI deduction, respectively? A) $105,000 and $21,000 B) $101,000 and $20,200 C) $105,000 and $20,011 D) $100,055 and $20,011

The correct answer is (D).Jerry's QBI from the partnership is $65,055, or 20% × $350,000 − $4,945. His QBI from the S corporation is $35,000, or 70% × $50,000. His total QBI is $100,055, or $65,055 + $35,000, and his QBI deduction is $20,011, or 20% × $100,055.

Vincent is a wealthy attorney in the highest marginal income tax bracket. He is interested in purchasing a franchise with some of his associates. After reviewing the proposal, you have determined that apart from a substantial up-front investment, the business will not need to retain income, and any income generated in subsequent years will be paid out to the investors. Vincent wants to be assured that the business would not be disrupted if one of his associates lost interest and wanted to leave the business. Vincent also has some concerns about personal liability that may arise in connection with the business and would like some protection in that respect. What form of business structure would make the most sense? A) A limited partnership B) A general partnership C) A C corporation D) An S corporation

The correct answer is (D).Options (A) and (B) are incorrect because both forms would dissolve in the event of a 50 percent turnover in ownership in a 12-month period. In addition, the general partnership provides no liability protection, and the limited partnership would only provide protection from liability if Vincent were a limited partner. Since he is making a substantial investment, it is likely he will desire some management control, making the limited partnership undesirable. Option (C) is incorrect because a C corporation generally retains income. Option (D) is correct as an S corporation has no retained earnings subject to tax since all profits and losses are passed through to its owners. S corporations offer limited liability and can remain operational even in cases in which shareholders leave, as an S corporation can be owned and operated by a single shareholder.


Set pelajaran terkait

CH. 16 - Managing Change & Stress

View Set

HIST201 - Module 1 - The Americas and Early Globalization (Before 1492-1650) (2 Weeks)

View Set

Acid-Base Balance Practice Questions

View Set

ElNEC Quizzes Module 2 WGU C475.

View Set

ECON 2411 Ch. 2 - An Overview of the Financial System

View Set

Chapter 16 study guide questions

View Set